2014 dxdy logo

Научный форум dxdy

Математика, Физика, Computer Science, Machine Learning, LaTeX, Механика и Техника, Химия,
Биология и Медицина, Экономика и Финансовая Математика, Гуманитарные науки


Правила форума


В этом разделе нельзя создавать новые темы.

Если Вы хотите задать новый вопрос, то не дописывайте его в существующую тему, а создайте новую в корневом разделе "Помогите решить/разобраться (М)".

Если Вы зададите новый вопрос в существующей теме, то в случае нарушения оформления или других правил форума Ваше сообщение и все ответы на него могут быть удалены без предупреждения.

Не ищите на этом форуме халяву, правила запрещают участникам публиковать готовые решения стандартных учебных задач. Автор вопроса обязан привести свои попытки решения и указать конкретные затруднения.

Обязательно просмотрите тему Правила данного раздела, иначе Ваша тема может быть удалена или перемещена в Карантин, а Вы так и не узнаете, почему.



Начать новую тему Ответить на тему
 
 помогите с интегралом
Сообщение16.04.2015, 21:46 


26/12/13
228
Здравствуйте, решая задачу о полноте достаточной статистики пришел к интегралу $\int\limits_{0}^{1} f(x)x^{\theta -1}  dx$

Необходимо показать, что для любого $\theta > 0$ интеграл равен нулю тогда и только тогда когда $f(x)=0$
Где-то слышал, что это следует из скалярного произведения, но очень бы не хотелось это использовать.
Я пытаюсь решить задачу следующим образом:
Рассматриваю пока просто случай, когда функция меняет знак только один раз в точке $a$.Тогда для любого разбиения отрезка $[0;a]$ на не пересекающиеся интервалы найдется разбиение отрезка $[a;1]$ такое что интегралу $\int\limits_{0}^{x1} f(x)x^{\theta -1}  dx$ найдется отрезок $\int\limits_{a}^{b1} f(x)x^{\theta -1}  dx$ что интегралы будут равны по модулю. Рассматриваю для $\theta=1$ и $\theta=2$ получаю 2 равенства
$\int\limits_{0}^{a} f(x)x  dx  - \int\limits_{a}^{1} f(x)x  dx=0$
$\int\limits_{0}^{a} f(x)  dx - \int\limits_{a}^{1} f(x)  dx =0$

Вроде бы исходя из них должно быть как-то очевидно, что $f(x)$ равна нулю, ведь я функцию в каждой точке умножаю на некий коээфицент меньше единицы, пробовал как-то складывать и вычитать эти равенства, но что-то не пришел к результату, подскажите пожалуйста

 Профиль  
                  
 
 Re: помогите с интегралом
Сообщение16.04.2015, 21:50 
Заслуженный участник


09/05/13
8904
$f$, надо думать, неотрицательна. Чисто по контексту. О чем Вы не упомянули. Иначе это просто неверно.
Если так, то это стандартный результат из теории меры и интеграла Лебега. Действительно, будет равна нулю почти всюду.

 Профиль  
                  
 
 Re: помогите с интегралом
Сообщение16.04.2015, 21:50 
Заслуженный участник
Аватара пользователя


22/01/11
2641
СПб
loshka в сообщении #1004561 писал(а):
интеграл равен нулю тогда и только тогда когда $f(x)=0$ для любого $\theta > 0$

вероятно, кванторы такие: интеграл равен нулю для любого $\theta>0$ в том и только в том случае, когда $f=0$

-- Чт апр 16, 2015 21:52:08 --

Otta в сообщении #1004564 писал(а):
$f$, надо думать, неотрицательна

с чего бы вдруг? Там же $\forall \theta>0$

 Профиль  
                  
 
 Re: помогите с интегралом
Сообщение16.04.2015, 21:53 
Заслуженный участник


09/05/13
8904
:D Что для любого $\theta$?

loshka, правда что, навесьте кванторы куда надо, а то так мы не договоримся.

 Профиль  
                  
 
 Re: помогите с интегралом
Сообщение16.04.2015, 21:55 


26/12/13
228
да, перепутал квантары, эм, так $f(x)$ может быть отрицательна, почему при таком раскладе это неверно?

 Профиль  
                  
 
 Re: помогите с интегралом
Сообщение16.04.2015, 22:00 
Заслуженный участник
Аватара пользователя


22/01/11
2641
СПб
loshka
для любого $\theta>1$ производная интеграла по $\theta$ равна нулю

 Профиль  
                  
 
 Re: помогите с интегралом
Сообщение16.04.2015, 22:11 


26/12/13
228
alcoholist

Эм, как-то я просто не вижу, что из того что $F'(\theta)=\int\limits_{0}^{1} f(x)x^{\theta -1}\operatorname{Ln}{(x)}  dx=0$ что-то следует :facepalm:

 Профиль  
                  
 
 Re: помогите с интегралом
Сообщение16.04.2015, 23:25 
Заслуженный участник
Аватара пользователя


11/12/05
9957
loshka в сообщении #1004570 писал(а):
да, перепутал квантары, эм, так $f(x)$ может быть отрицательна, почему при таком раскладе это неверно?
$\theta = 1>0; \quad  f(x) = (x-0.5) \not\equiv  0$

 Профиль  
                  
 
 Re: помогите с интегралом
Сообщение16.04.2015, 23:38 


26/12/13
228
я вообще условие полноты написал мягко говоря написал не правильно, надо перестать заниматься ерундой и самому искать ответы на простые вопросы

 Профиль  
                  
 
 Re: помогите с интегралом
Сообщение16.04.2015, 23:44 
Заслуженный участник


09/05/13
8904
Вы кванторы расставили бестолково, отсюда и все непонятки пошли. Самому, конечно, лучше.
А так - это ж преобразование Меллина. Ну и воспользуйтесь его свойствами.

 Профиль  
                  
 
 Re: помогите с интегралом
Сообщение16.04.2015, 23:48 
Заслуженный участник
Аватара пользователя


01/03/06
13626
Москва
И все-таки, почему нужно отказаться от стандартной идеи полных систем? Из условия задачи получается, что, скажем, в $L_2[0 ; 1]$ исследуемая функция ортогональна полной системе соответствующим образом модифицированных многочленов Лежандра, и тогда рассматриваемая функция должна быть нулем в этом пространстве.

 Профиль  
                  
 
 Re: помогите с интегралом
Сообщение16.04.2015, 23:51 
Заслуженный участник


09/05/13
8904
:) Так совсем лепота. :D

 Профиль  
                  
Показать сообщения за:  Поле сортировки  
Начать новую тему Ответить на тему  [ Сообщений: 12 ] 

Модераторы: Модераторы Математики, Супермодераторы



Кто сейчас на конференции

Сейчас этот форум просматривают: нет зарегистрированных пользователей


Вы не можете начинать темы
Вы не можете отвечать на сообщения
Вы не можете редактировать свои сообщения
Вы не можете удалять свои сообщения
Вы не можете добавлять вложения

Найти:
Powered by phpBB © 2000, 2002, 2005, 2007 phpBB Group